How do I find all prime solutions $p, q, r$ of the equation $displaystyle p(p+1)+q(q+1) = r(r+1)$?











up vote
5
down vote

favorite
2













Find primes $p, q, r$ of the equation $$p(p+1)+q(q+1)
= r(r+1)$$




I know that it has only one solution namely $p = q = 2,r = 3$. But i can't show that.



Thank you for any help










share|cite|improve this question
























  • How do you know that no other solutions exist then, what is your source ?
    – Peter
    Dec 12 '16 at 22:28










  • I think this problem related to A shinzel solution of the titled equation if i'm true
    – zeraoulia rafik
    Dec 12 '16 at 22:43










  • It may or may not help to rephrase it as $p^2 + p + q^2 + q = r^2 + r$.
    – Robert Soupe
    Dec 13 '16 at 2:21












  • this equivalent to solve $$(2p+1)^2+(2q+1)^2=(2r+1)^2+1$$
    – Bumblebee
    Jan 18 at 20:01















up vote
5
down vote

favorite
2













Find primes $p, q, r$ of the equation $$p(p+1)+q(q+1)
= r(r+1)$$




I know that it has only one solution namely $p = q = 2,r = 3$. But i can't show that.



Thank you for any help










share|cite|improve this question
























  • How do you know that no other solutions exist then, what is your source ?
    – Peter
    Dec 12 '16 at 22:28










  • I think this problem related to A shinzel solution of the titled equation if i'm true
    – zeraoulia rafik
    Dec 12 '16 at 22:43










  • It may or may not help to rephrase it as $p^2 + p + q^2 + q = r^2 + r$.
    – Robert Soupe
    Dec 13 '16 at 2:21












  • this equivalent to solve $$(2p+1)^2+(2q+1)^2=(2r+1)^2+1$$
    – Bumblebee
    Jan 18 at 20:01













up vote
5
down vote

favorite
2









up vote
5
down vote

favorite
2






2






Find primes $p, q, r$ of the equation $$p(p+1)+q(q+1)
= r(r+1)$$




I know that it has only one solution namely $p = q = 2,r = 3$. But i can't show that.



Thank you for any help










share|cite|improve this question
















Find primes $p, q, r$ of the equation $$p(p+1)+q(q+1)
= r(r+1)$$




I know that it has only one solution namely $p = q = 2,r = 3$. But i can't show that.



Thank you for any help







number-theory elementary-number-theory prime-numbers






share|cite|improve this question















share|cite|improve this question













share|cite|improve this question




share|cite|improve this question








edited Nov 15 at 17:25









greedoid

34.5k114489




34.5k114489










asked Dec 12 '16 at 22:12









Youssra El Yossra Youssra

975




975












  • How do you know that no other solutions exist then, what is your source ?
    – Peter
    Dec 12 '16 at 22:28










  • I think this problem related to A shinzel solution of the titled equation if i'm true
    – zeraoulia rafik
    Dec 12 '16 at 22:43










  • It may or may not help to rephrase it as $p^2 + p + q^2 + q = r^2 + r$.
    – Robert Soupe
    Dec 13 '16 at 2:21












  • this equivalent to solve $$(2p+1)^2+(2q+1)^2=(2r+1)^2+1$$
    – Bumblebee
    Jan 18 at 20:01


















  • How do you know that no other solutions exist then, what is your source ?
    – Peter
    Dec 12 '16 at 22:28










  • I think this problem related to A shinzel solution of the titled equation if i'm true
    – zeraoulia rafik
    Dec 12 '16 at 22:43










  • It may or may not help to rephrase it as $p^2 + p + q^2 + q = r^2 + r$.
    – Robert Soupe
    Dec 13 '16 at 2:21












  • this equivalent to solve $$(2p+1)^2+(2q+1)^2=(2r+1)^2+1$$
    – Bumblebee
    Jan 18 at 20:01
















How do you know that no other solutions exist then, what is your source ?
– Peter
Dec 12 '16 at 22:28




How do you know that no other solutions exist then, what is your source ?
– Peter
Dec 12 '16 at 22:28












I think this problem related to A shinzel solution of the titled equation if i'm true
– zeraoulia rafik
Dec 12 '16 at 22:43




I think this problem related to A shinzel solution of the titled equation if i'm true
– zeraoulia rafik
Dec 12 '16 at 22:43












It may or may not help to rephrase it as $p^2 + p + q^2 + q = r^2 + r$.
– Robert Soupe
Dec 13 '16 at 2:21






It may or may not help to rephrase it as $p^2 + p + q^2 + q = r^2 + r$.
– Robert Soupe
Dec 13 '16 at 2:21














this equivalent to solve $$(2p+1)^2+(2q+1)^2=(2r+1)^2+1$$
– Bumblebee
Jan 18 at 20:01




this equivalent to solve $$(2p+1)^2+(2q+1)^2=(2r+1)^2+1$$
– Bumblebee
Jan 18 at 20:01










3 Answers
3






active

oldest

votes

















up vote
7
down vote



accepted










May this lead to a simple proof for your problem according to your unic example of solution . There is only one
solution, namely $p = q = 2,r = 3$. To see that, we shall find all solutions
of the equation $p(p+1)+q(q+1) = n(n+1)$ where $p$ and $q$ are primes and
$n$ is a positive integer. Our equation yields
$p(p+1) = n(n+1)-q(q+1) = (n-q)(n+q+1)$,
and we must have $n > q$. Since $p$ is a prime, we have either $p|n-q$ or
$p|n+q+1$. If $p|n-q$, then we have $pleq n-q$, which implies $p(p+1)
leq (n-q)(n-q+1)$, and therefore $n+q+1 leq n-q+1$, which is impossible.
Thus we have $p|n+q+ 1$, which means that for some positive integer $k$
,$n+q+1 = kp$, which implies $p+1 = k(n-q)tag1$.



If we had $k = 1$, then $n+q+ 1 = p$ and $p+ 1 = n-q$, which gives $p-q
= n+ 1$ and $p+q = n- 1$, which is impossible, because $(p+q)-(p-q)=2q>0$ and $(n-1)-(n+1)=-2<0$. Thus, $k > 1$. From $(1)$ we
easily obtain:



$2q = (n+q)-(n-q) = kp-1-(n-q)
= k[k(n-q)-1]-1-(n-q) = (k+1)[(k-1)(n-q)-1]$



.Since $k geq 2$, we have $k+1 geq 3$. The last equality, whose left-hand side has
positive integer divisors $1, 2, q$, and $2q$ only, implies that either $k+ 1 = q$
or $k+1 = 2q$. If $k+1 = q$, then $(k-1)(n-q) = 3$, hence $(q-2)(n-q) = 3$.
This leads to either $q-2 = 1$, $n-q = 3$, that is $q = 3, n = 6, k = q-1 = 2$,
and, in view of $(1)$, $P = 5$, or else, $q-2 = 3$, $n-q = 1$, which gives $q = 5,
n = 6, k = 4$, and in view of $(1)$, $p = 3$.
On the other hand, if $k+1 = 2q$, then $(k-1)(n-q) = 2$, hence
$2(q-1)(n-q) = 2$. This leads to $q-1 = 1$ and $n-q = 1$, or $q = 2, n = 3$,
and, in view of $(1)$, $p = 2$. Thus, for positive integer $ n$, we have the
following solutions in primes $p$ and $q$: $(p = q = 2, n = 3; 2)$,$ (p = 5,
q = 3, n = 6)$, and $3$ ,$(p = 3, q = 5, n = 6)$. Only in the first solution all
three numbers are primes.



Note: If we denote by $displaystyle t_n = frac{n(n+1)}{2}$ the nth triangular number,
then the equation $t_p+t_q = t_r$
has only one solution in prime numbers, namely $p = q = 2, r = 3$.






share|cite|improve this answer























  • It should be $p+q=n-1$.
    – user236182
    Dec 12 '16 at 22:51










  • It's still impossible because $(p+q)-(p-q)=2q>0$ while $(n-1)-(n+1)=-2<0$.
    – user236182
    Dec 12 '16 at 22:54










  • for k=1 it's impossible and i cited your case p+q=n-1 in the side of n+q=p-1
    – zeraoulia rafik
    Dec 12 '16 at 22:55










  • If $p+1=n-q$, then $p+q=n-1$.
    – user236182
    Dec 12 '16 at 22:57










  • And if $n+q+1=p$, then $p-q=n+1$.
    – user236182
    Dec 12 '16 at 22:58


















up vote
0
down vote













Case $pne q$: We can assume that $p>q$, then $$r^2+r leq (p-1)^2+p^2+p-1+p=2p^2implies boxed{rleq psqrt{2}}$$



From:



$$p(p+1) =r^2- q^2+r-q =(r-q)(r+q+1)$$
we get



If $pmid r-q$ then $r+q+1mid p+1$, so $pleq r-q<r$ and $r+q+1leq p+1 implies r<p$ a contradiction.



If $pmid r+q+1$ then $r-qmid p+1$. Since $r+q+1=kp$ for some integer $kgeq 1$ we have $$kpleq psqrt{2}+(p-1)+1 implies kleq 2$$



Case 1: $k=2$ we get $r+q+1=2p$ and $p+1=2r-2q$ from where we get $3p=11$, no good.



Case 2: $k=1$ we get $r+q+1=p$ and $p+1=r-q$ so $q=-1$ again contradiction.





So $p=q$ and now we have to solve $$2p^2+2p = r^2+r$$



So $$2p(p+1)=r(r+1)implies rmid 2p(p+1)$$



If $rmid 2$ then $r=2$ which is impossibile.



If $rmid p$ then $rleq p$ which is impossibile.



If $rmid p+1$ then $rleq p+1$ but then $r=p+1$ since $r>p$, so $r=3$ and $p=2$.






share|cite|improve this answer




























    up vote
    0
    down vote













    From the given equation
    $$p(p+1)+q(q+1)=r(r+1)$$
    it follows that $p < r$ and $q < r$.



    Next, another inequality which will be useful later . . .



    Claim:$;p+q > r$.



    Proof:
    begin{align*}
    &p(p+1)+q(q+1)=r(r+1)\[4pt]
    implies;&(p^2+q^2)+(p+q)=r(r+1)\[4pt]
    implies;&(p+q)^2+(p+q)>r(r+1)\[4pt]
    implies;&(p+q)(p+q+1)>r(r+1)\[4pt]
    implies;&p+q>r\[4pt]
    end{align*}

    as claimed.



    Returning to the main problem . . .



    First suppose $p=q$.



    Then the given equation reduces to
    $$2p(p+1)=r(r+1)$$
    hence, since $r > p$, it follows that $r|(p+1)$.



    But then $p < r le p+1$, so $r=p+1$, which implies $p=2$, and $r=3$.



    It can be verified that the triple $(p,q,r)=(2,2,3)$ satisfies the given equation.



    Next suppose $p,q$ are distinct.



    Without loss of generality, assume $p < q$.



    Suppose $;p=2$.



    Then from $p < q < r$, we get $qge 3$ and $rge 5$, hence
    begin{align*}
    &p(p+1)+q(q+1)=r(r+1)\[4pt]
    implies;&(2)(2+1)+q(q+1)=r(r+1)\[4pt]
    implies;&r(r+1)-q(q+1)=6\[4pt]
    implies;&(r-q)(q+r+1)=6\[4pt]
    implies;&(q+r+1)mid 6\[4pt]
    implies;&q+r+1le 6\[4pt]
    end{align*}

    contradiction, since $q+r+1ge 3+5+1=9$.



    Hence we must have $p > 2$.



    Since $p+q > r$, it follows that $pnotmid r-q$, and $qnotmid r-p$.
    begin{align*}
    text{Then};;&p(p+1)+q(q+1)=r(r+1)\[4pt]
    implies;&p(p+1)=r(r+1)-q(q+1)\[4pt]
    implies;&p(p+1)=(q+r+1)(r-q)\[4pt]
    implies;&pmid (q+r+1)\[4pt]
    implies;&pmid (p+q+r+1)\[12pt]
    text{and};,&p(p+1)+q(q+1)=r(r+1)\[4pt]
    implies;&q(q+1)=r(r+1)-p(p+1)\[4pt]
    implies;&q(q+1)=(p+r+1)(r-p)\[4pt]
    implies;&qmid (p+r+1)\[4pt]
    implies;&qmid (p+q+r+1)\[12pt]
    text{hence};,&pqmid (p+q+r+1)\[4pt]
    implies;&pqle p+q+r+1\[4pt]
    implies;&pq < p+q+(p+q)+1\[4pt]
    implies;&pq-2p-2q < 1\[4pt]
    implies;&(p-2)(q-2) < 5\[4pt]
    implies;&q-2 < 5\[4pt]
    implies;&q < 7\[4pt]
    implies;&qle 5\[4pt]
    implies;&(p,q)=(3,5)\[4pt]
    implies;&(3)(3+1)+(5)(5+1)=r(r+1)\[4pt]
    implies;&r=6\[4pt]
    end{align*}

    contradiction, since $6$ is not prime.



    Therefore the only solution is $(p,q,r)=(2,2,3)$.






    share|cite|improve this answer























      Your Answer





      StackExchange.ifUsing("editor", function () {
      return StackExchange.using("mathjaxEditing", function () {
      StackExchange.MarkdownEditor.creationCallbacks.add(function (editor, postfix) {
      StackExchange.mathjaxEditing.prepareWmdForMathJax(editor, postfix, [["$", "$"], ["\\(","\\)"]]);
      });
      });
      }, "mathjax-editing");

      StackExchange.ready(function() {
      var channelOptions = {
      tags: "".split(" "),
      id: "69"
      };
      initTagRenderer("".split(" "), "".split(" "), channelOptions);

      StackExchange.using("externalEditor", function() {
      // Have to fire editor after snippets, if snippets enabled
      if (StackExchange.settings.snippets.snippetsEnabled) {
      StackExchange.using("snippets", function() {
      createEditor();
      });
      }
      else {
      createEditor();
      }
      });

      function createEditor() {
      StackExchange.prepareEditor({
      heartbeatType: 'answer',
      convertImagesToLinks: true,
      noModals: true,
      showLowRepImageUploadWarning: true,
      reputationToPostImages: 10,
      bindNavPrevention: true,
      postfix: "",
      imageUploader: {
      brandingHtml: "Powered by u003ca class="icon-imgur-white" href="https://imgur.com/"u003eu003c/au003e",
      contentPolicyHtml: "User contributions licensed under u003ca href="https://creativecommons.org/licenses/by-sa/3.0/"u003ecc by-sa 3.0 with attribution requiredu003c/au003e u003ca href="https://stackoverflow.com/legal/content-policy"u003e(content policy)u003c/au003e",
      allowUrls: true
      },
      noCode: true, onDemand: true,
      discardSelector: ".discard-answer"
      ,immediatelyShowMarkdownHelp:true
      });


      }
      });














       

      draft saved


      draft discarded


















      StackExchange.ready(
      function () {
      StackExchange.openid.initPostLogin('.new-post-login', 'https%3a%2f%2fmath.stackexchange.com%2fquestions%2f2056120%2fhow-do-i-find-all-prime-solutions-p-q-r-of-the-equation-displaystyle-pp1%23new-answer', 'question_page');
      }
      );

      Post as a guest















      Required, but never shown

























      3 Answers
      3






      active

      oldest

      votes








      3 Answers
      3






      active

      oldest

      votes









      active

      oldest

      votes






      active

      oldest

      votes








      up vote
      7
      down vote



      accepted










      May this lead to a simple proof for your problem according to your unic example of solution . There is only one
      solution, namely $p = q = 2,r = 3$. To see that, we shall find all solutions
      of the equation $p(p+1)+q(q+1) = n(n+1)$ where $p$ and $q$ are primes and
      $n$ is a positive integer. Our equation yields
      $p(p+1) = n(n+1)-q(q+1) = (n-q)(n+q+1)$,
      and we must have $n > q$. Since $p$ is a prime, we have either $p|n-q$ or
      $p|n+q+1$. If $p|n-q$, then we have $pleq n-q$, which implies $p(p+1)
      leq (n-q)(n-q+1)$, and therefore $n+q+1 leq n-q+1$, which is impossible.
      Thus we have $p|n+q+ 1$, which means that for some positive integer $k$
      ,$n+q+1 = kp$, which implies $p+1 = k(n-q)tag1$.



      If we had $k = 1$, then $n+q+ 1 = p$ and $p+ 1 = n-q$, which gives $p-q
      = n+ 1$ and $p+q = n- 1$, which is impossible, because $(p+q)-(p-q)=2q>0$ and $(n-1)-(n+1)=-2<0$. Thus, $k > 1$. From $(1)$ we
      easily obtain:



      $2q = (n+q)-(n-q) = kp-1-(n-q)
      = k[k(n-q)-1]-1-(n-q) = (k+1)[(k-1)(n-q)-1]$



      .Since $k geq 2$, we have $k+1 geq 3$. The last equality, whose left-hand side has
      positive integer divisors $1, 2, q$, and $2q$ only, implies that either $k+ 1 = q$
      or $k+1 = 2q$. If $k+1 = q$, then $(k-1)(n-q) = 3$, hence $(q-2)(n-q) = 3$.
      This leads to either $q-2 = 1$, $n-q = 3$, that is $q = 3, n = 6, k = q-1 = 2$,
      and, in view of $(1)$, $P = 5$, or else, $q-2 = 3$, $n-q = 1$, which gives $q = 5,
      n = 6, k = 4$, and in view of $(1)$, $p = 3$.
      On the other hand, if $k+1 = 2q$, then $(k-1)(n-q) = 2$, hence
      $2(q-1)(n-q) = 2$. This leads to $q-1 = 1$ and $n-q = 1$, or $q = 2, n = 3$,
      and, in view of $(1)$, $p = 2$. Thus, for positive integer $ n$, we have the
      following solutions in primes $p$ and $q$: $(p = q = 2, n = 3; 2)$,$ (p = 5,
      q = 3, n = 6)$, and $3$ ,$(p = 3, q = 5, n = 6)$. Only in the first solution all
      three numbers are primes.



      Note: If we denote by $displaystyle t_n = frac{n(n+1)}{2}$ the nth triangular number,
      then the equation $t_p+t_q = t_r$
      has only one solution in prime numbers, namely $p = q = 2, r = 3$.






      share|cite|improve this answer























      • It should be $p+q=n-1$.
        – user236182
        Dec 12 '16 at 22:51










      • It's still impossible because $(p+q)-(p-q)=2q>0$ while $(n-1)-(n+1)=-2<0$.
        – user236182
        Dec 12 '16 at 22:54










      • for k=1 it's impossible and i cited your case p+q=n-1 in the side of n+q=p-1
        – zeraoulia rafik
        Dec 12 '16 at 22:55










      • If $p+1=n-q$, then $p+q=n-1$.
        – user236182
        Dec 12 '16 at 22:57










      • And if $n+q+1=p$, then $p-q=n+1$.
        – user236182
        Dec 12 '16 at 22:58















      up vote
      7
      down vote



      accepted










      May this lead to a simple proof for your problem according to your unic example of solution . There is only one
      solution, namely $p = q = 2,r = 3$. To see that, we shall find all solutions
      of the equation $p(p+1)+q(q+1) = n(n+1)$ where $p$ and $q$ are primes and
      $n$ is a positive integer. Our equation yields
      $p(p+1) = n(n+1)-q(q+1) = (n-q)(n+q+1)$,
      and we must have $n > q$. Since $p$ is a prime, we have either $p|n-q$ or
      $p|n+q+1$. If $p|n-q$, then we have $pleq n-q$, which implies $p(p+1)
      leq (n-q)(n-q+1)$, and therefore $n+q+1 leq n-q+1$, which is impossible.
      Thus we have $p|n+q+ 1$, which means that for some positive integer $k$
      ,$n+q+1 = kp$, which implies $p+1 = k(n-q)tag1$.



      If we had $k = 1$, then $n+q+ 1 = p$ and $p+ 1 = n-q$, which gives $p-q
      = n+ 1$ and $p+q = n- 1$, which is impossible, because $(p+q)-(p-q)=2q>0$ and $(n-1)-(n+1)=-2<0$. Thus, $k > 1$. From $(1)$ we
      easily obtain:



      $2q = (n+q)-(n-q) = kp-1-(n-q)
      = k[k(n-q)-1]-1-(n-q) = (k+1)[(k-1)(n-q)-1]$



      .Since $k geq 2$, we have $k+1 geq 3$. The last equality, whose left-hand side has
      positive integer divisors $1, 2, q$, and $2q$ only, implies that either $k+ 1 = q$
      or $k+1 = 2q$. If $k+1 = q$, then $(k-1)(n-q) = 3$, hence $(q-2)(n-q) = 3$.
      This leads to either $q-2 = 1$, $n-q = 3$, that is $q = 3, n = 6, k = q-1 = 2$,
      and, in view of $(1)$, $P = 5$, or else, $q-2 = 3$, $n-q = 1$, which gives $q = 5,
      n = 6, k = 4$, and in view of $(1)$, $p = 3$.
      On the other hand, if $k+1 = 2q$, then $(k-1)(n-q) = 2$, hence
      $2(q-1)(n-q) = 2$. This leads to $q-1 = 1$ and $n-q = 1$, or $q = 2, n = 3$,
      and, in view of $(1)$, $p = 2$. Thus, for positive integer $ n$, we have the
      following solutions in primes $p$ and $q$: $(p = q = 2, n = 3; 2)$,$ (p = 5,
      q = 3, n = 6)$, and $3$ ,$(p = 3, q = 5, n = 6)$. Only in the first solution all
      three numbers are primes.



      Note: If we denote by $displaystyle t_n = frac{n(n+1)}{2}$ the nth triangular number,
      then the equation $t_p+t_q = t_r$
      has only one solution in prime numbers, namely $p = q = 2, r = 3$.






      share|cite|improve this answer























      • It should be $p+q=n-1$.
        – user236182
        Dec 12 '16 at 22:51










      • It's still impossible because $(p+q)-(p-q)=2q>0$ while $(n-1)-(n+1)=-2<0$.
        – user236182
        Dec 12 '16 at 22:54










      • for k=1 it's impossible and i cited your case p+q=n-1 in the side of n+q=p-1
        – zeraoulia rafik
        Dec 12 '16 at 22:55










      • If $p+1=n-q$, then $p+q=n-1$.
        – user236182
        Dec 12 '16 at 22:57










      • And if $n+q+1=p$, then $p-q=n+1$.
        – user236182
        Dec 12 '16 at 22:58













      up vote
      7
      down vote



      accepted







      up vote
      7
      down vote



      accepted






      May this lead to a simple proof for your problem according to your unic example of solution . There is only one
      solution, namely $p = q = 2,r = 3$. To see that, we shall find all solutions
      of the equation $p(p+1)+q(q+1) = n(n+1)$ where $p$ and $q$ are primes and
      $n$ is a positive integer. Our equation yields
      $p(p+1) = n(n+1)-q(q+1) = (n-q)(n+q+1)$,
      and we must have $n > q$. Since $p$ is a prime, we have either $p|n-q$ or
      $p|n+q+1$. If $p|n-q$, then we have $pleq n-q$, which implies $p(p+1)
      leq (n-q)(n-q+1)$, and therefore $n+q+1 leq n-q+1$, which is impossible.
      Thus we have $p|n+q+ 1$, which means that for some positive integer $k$
      ,$n+q+1 = kp$, which implies $p+1 = k(n-q)tag1$.



      If we had $k = 1$, then $n+q+ 1 = p$ and $p+ 1 = n-q$, which gives $p-q
      = n+ 1$ and $p+q = n- 1$, which is impossible, because $(p+q)-(p-q)=2q>0$ and $(n-1)-(n+1)=-2<0$. Thus, $k > 1$. From $(1)$ we
      easily obtain:



      $2q = (n+q)-(n-q) = kp-1-(n-q)
      = k[k(n-q)-1]-1-(n-q) = (k+1)[(k-1)(n-q)-1]$



      .Since $k geq 2$, we have $k+1 geq 3$. The last equality, whose left-hand side has
      positive integer divisors $1, 2, q$, and $2q$ only, implies that either $k+ 1 = q$
      or $k+1 = 2q$. If $k+1 = q$, then $(k-1)(n-q) = 3$, hence $(q-2)(n-q) = 3$.
      This leads to either $q-2 = 1$, $n-q = 3$, that is $q = 3, n = 6, k = q-1 = 2$,
      and, in view of $(1)$, $P = 5$, or else, $q-2 = 3$, $n-q = 1$, which gives $q = 5,
      n = 6, k = 4$, and in view of $(1)$, $p = 3$.
      On the other hand, if $k+1 = 2q$, then $(k-1)(n-q) = 2$, hence
      $2(q-1)(n-q) = 2$. This leads to $q-1 = 1$ and $n-q = 1$, or $q = 2, n = 3$,
      and, in view of $(1)$, $p = 2$. Thus, for positive integer $ n$, we have the
      following solutions in primes $p$ and $q$: $(p = q = 2, n = 3; 2)$,$ (p = 5,
      q = 3, n = 6)$, and $3$ ,$(p = 3, q = 5, n = 6)$. Only in the first solution all
      three numbers are primes.



      Note: If we denote by $displaystyle t_n = frac{n(n+1)}{2}$ the nth triangular number,
      then the equation $t_p+t_q = t_r$
      has only one solution in prime numbers, namely $p = q = 2, r = 3$.






      share|cite|improve this answer














      May this lead to a simple proof for your problem according to your unic example of solution . There is only one
      solution, namely $p = q = 2,r = 3$. To see that, we shall find all solutions
      of the equation $p(p+1)+q(q+1) = n(n+1)$ where $p$ and $q$ are primes and
      $n$ is a positive integer. Our equation yields
      $p(p+1) = n(n+1)-q(q+1) = (n-q)(n+q+1)$,
      and we must have $n > q$. Since $p$ is a prime, we have either $p|n-q$ or
      $p|n+q+1$. If $p|n-q$, then we have $pleq n-q$, which implies $p(p+1)
      leq (n-q)(n-q+1)$, and therefore $n+q+1 leq n-q+1$, which is impossible.
      Thus we have $p|n+q+ 1$, which means that for some positive integer $k$
      ,$n+q+1 = kp$, which implies $p+1 = k(n-q)tag1$.



      If we had $k = 1$, then $n+q+ 1 = p$ and $p+ 1 = n-q$, which gives $p-q
      = n+ 1$ and $p+q = n- 1$, which is impossible, because $(p+q)-(p-q)=2q>0$ and $(n-1)-(n+1)=-2<0$. Thus, $k > 1$. From $(1)$ we
      easily obtain:



      $2q = (n+q)-(n-q) = kp-1-(n-q)
      = k[k(n-q)-1]-1-(n-q) = (k+1)[(k-1)(n-q)-1]$



      .Since $k geq 2$, we have $k+1 geq 3$. The last equality, whose left-hand side has
      positive integer divisors $1, 2, q$, and $2q$ only, implies that either $k+ 1 = q$
      or $k+1 = 2q$. If $k+1 = q$, then $(k-1)(n-q) = 3$, hence $(q-2)(n-q) = 3$.
      This leads to either $q-2 = 1$, $n-q = 3$, that is $q = 3, n = 6, k = q-1 = 2$,
      and, in view of $(1)$, $P = 5$, or else, $q-2 = 3$, $n-q = 1$, which gives $q = 5,
      n = 6, k = 4$, and in view of $(1)$, $p = 3$.
      On the other hand, if $k+1 = 2q$, then $(k-1)(n-q) = 2$, hence
      $2(q-1)(n-q) = 2$. This leads to $q-1 = 1$ and $n-q = 1$, or $q = 2, n = 3$,
      and, in view of $(1)$, $p = 2$. Thus, for positive integer $ n$, we have the
      following solutions in primes $p$ and $q$: $(p = q = 2, n = 3; 2)$,$ (p = 5,
      q = 3, n = 6)$, and $3$ ,$(p = 3, q = 5, n = 6)$. Only in the first solution all
      three numbers are primes.



      Note: If we denote by $displaystyle t_n = frac{n(n+1)}{2}$ the nth triangular number,
      then the equation $t_p+t_q = t_r$
      has only one solution in prime numbers, namely $p = q = 2, r = 3$.







      share|cite|improve this answer














      share|cite|improve this answer



      share|cite|improve this answer








      edited Dec 12 '16 at 23:23









      user236182

      11.9k11233




      11.9k11233










      answered Dec 12 '16 at 22:46









      zeraoulia rafik

      2,30911029




      2,30911029












      • It should be $p+q=n-1$.
        – user236182
        Dec 12 '16 at 22:51










      • It's still impossible because $(p+q)-(p-q)=2q>0$ while $(n-1)-(n+1)=-2<0$.
        – user236182
        Dec 12 '16 at 22:54










      • for k=1 it's impossible and i cited your case p+q=n-1 in the side of n+q=p-1
        – zeraoulia rafik
        Dec 12 '16 at 22:55










      • If $p+1=n-q$, then $p+q=n-1$.
        – user236182
        Dec 12 '16 at 22:57










      • And if $n+q+1=p$, then $p-q=n+1$.
        – user236182
        Dec 12 '16 at 22:58


















      • It should be $p+q=n-1$.
        – user236182
        Dec 12 '16 at 22:51










      • It's still impossible because $(p+q)-(p-q)=2q>0$ while $(n-1)-(n+1)=-2<0$.
        – user236182
        Dec 12 '16 at 22:54










      • for k=1 it's impossible and i cited your case p+q=n-1 in the side of n+q=p-1
        – zeraoulia rafik
        Dec 12 '16 at 22:55










      • If $p+1=n-q$, then $p+q=n-1$.
        – user236182
        Dec 12 '16 at 22:57










      • And if $n+q+1=p$, then $p-q=n+1$.
        – user236182
        Dec 12 '16 at 22:58
















      It should be $p+q=n-1$.
      – user236182
      Dec 12 '16 at 22:51




      It should be $p+q=n-1$.
      – user236182
      Dec 12 '16 at 22:51












      It's still impossible because $(p+q)-(p-q)=2q>0$ while $(n-1)-(n+1)=-2<0$.
      – user236182
      Dec 12 '16 at 22:54




      It's still impossible because $(p+q)-(p-q)=2q>0$ while $(n-1)-(n+1)=-2<0$.
      – user236182
      Dec 12 '16 at 22:54












      for k=1 it's impossible and i cited your case p+q=n-1 in the side of n+q=p-1
      – zeraoulia rafik
      Dec 12 '16 at 22:55




      for k=1 it's impossible and i cited your case p+q=n-1 in the side of n+q=p-1
      – zeraoulia rafik
      Dec 12 '16 at 22:55












      If $p+1=n-q$, then $p+q=n-1$.
      – user236182
      Dec 12 '16 at 22:57




      If $p+1=n-q$, then $p+q=n-1$.
      – user236182
      Dec 12 '16 at 22:57












      And if $n+q+1=p$, then $p-q=n+1$.
      – user236182
      Dec 12 '16 at 22:58




      And if $n+q+1=p$, then $p-q=n+1$.
      – user236182
      Dec 12 '16 at 22:58










      up vote
      0
      down vote













      Case $pne q$: We can assume that $p>q$, then $$r^2+r leq (p-1)^2+p^2+p-1+p=2p^2implies boxed{rleq psqrt{2}}$$



      From:



      $$p(p+1) =r^2- q^2+r-q =(r-q)(r+q+1)$$
      we get



      If $pmid r-q$ then $r+q+1mid p+1$, so $pleq r-q<r$ and $r+q+1leq p+1 implies r<p$ a contradiction.



      If $pmid r+q+1$ then $r-qmid p+1$. Since $r+q+1=kp$ for some integer $kgeq 1$ we have $$kpleq psqrt{2}+(p-1)+1 implies kleq 2$$



      Case 1: $k=2$ we get $r+q+1=2p$ and $p+1=2r-2q$ from where we get $3p=11$, no good.



      Case 2: $k=1$ we get $r+q+1=p$ and $p+1=r-q$ so $q=-1$ again contradiction.





      So $p=q$ and now we have to solve $$2p^2+2p = r^2+r$$



      So $$2p(p+1)=r(r+1)implies rmid 2p(p+1)$$



      If $rmid 2$ then $r=2$ which is impossibile.



      If $rmid p$ then $rleq p$ which is impossibile.



      If $rmid p+1$ then $rleq p+1$ but then $r=p+1$ since $r>p$, so $r=3$ and $p=2$.






      share|cite|improve this answer

























        up vote
        0
        down vote













        Case $pne q$: We can assume that $p>q$, then $$r^2+r leq (p-1)^2+p^2+p-1+p=2p^2implies boxed{rleq psqrt{2}}$$



        From:



        $$p(p+1) =r^2- q^2+r-q =(r-q)(r+q+1)$$
        we get



        If $pmid r-q$ then $r+q+1mid p+1$, so $pleq r-q<r$ and $r+q+1leq p+1 implies r<p$ a contradiction.



        If $pmid r+q+1$ then $r-qmid p+1$. Since $r+q+1=kp$ for some integer $kgeq 1$ we have $$kpleq psqrt{2}+(p-1)+1 implies kleq 2$$



        Case 1: $k=2$ we get $r+q+1=2p$ and $p+1=2r-2q$ from where we get $3p=11$, no good.



        Case 2: $k=1$ we get $r+q+1=p$ and $p+1=r-q$ so $q=-1$ again contradiction.





        So $p=q$ and now we have to solve $$2p^2+2p = r^2+r$$



        So $$2p(p+1)=r(r+1)implies rmid 2p(p+1)$$



        If $rmid 2$ then $r=2$ which is impossibile.



        If $rmid p$ then $rleq p$ which is impossibile.



        If $rmid p+1$ then $rleq p+1$ but then $r=p+1$ since $r>p$, so $r=3$ and $p=2$.






        share|cite|improve this answer























          up vote
          0
          down vote










          up vote
          0
          down vote









          Case $pne q$: We can assume that $p>q$, then $$r^2+r leq (p-1)^2+p^2+p-1+p=2p^2implies boxed{rleq psqrt{2}}$$



          From:



          $$p(p+1) =r^2- q^2+r-q =(r-q)(r+q+1)$$
          we get



          If $pmid r-q$ then $r+q+1mid p+1$, so $pleq r-q<r$ and $r+q+1leq p+1 implies r<p$ a contradiction.



          If $pmid r+q+1$ then $r-qmid p+1$. Since $r+q+1=kp$ for some integer $kgeq 1$ we have $$kpleq psqrt{2}+(p-1)+1 implies kleq 2$$



          Case 1: $k=2$ we get $r+q+1=2p$ and $p+1=2r-2q$ from where we get $3p=11$, no good.



          Case 2: $k=1$ we get $r+q+1=p$ and $p+1=r-q$ so $q=-1$ again contradiction.





          So $p=q$ and now we have to solve $$2p^2+2p = r^2+r$$



          So $$2p(p+1)=r(r+1)implies rmid 2p(p+1)$$



          If $rmid 2$ then $r=2$ which is impossibile.



          If $rmid p$ then $rleq p$ which is impossibile.



          If $rmid p+1$ then $rleq p+1$ but then $r=p+1$ since $r>p$, so $r=3$ and $p=2$.






          share|cite|improve this answer












          Case $pne q$: We can assume that $p>q$, then $$r^2+r leq (p-1)^2+p^2+p-1+p=2p^2implies boxed{rleq psqrt{2}}$$



          From:



          $$p(p+1) =r^2- q^2+r-q =(r-q)(r+q+1)$$
          we get



          If $pmid r-q$ then $r+q+1mid p+1$, so $pleq r-q<r$ and $r+q+1leq p+1 implies r<p$ a contradiction.



          If $pmid r+q+1$ then $r-qmid p+1$. Since $r+q+1=kp$ for some integer $kgeq 1$ we have $$kpleq psqrt{2}+(p-1)+1 implies kleq 2$$



          Case 1: $k=2$ we get $r+q+1=2p$ and $p+1=2r-2q$ from where we get $3p=11$, no good.



          Case 2: $k=1$ we get $r+q+1=p$ and $p+1=r-q$ so $q=-1$ again contradiction.





          So $p=q$ and now we have to solve $$2p^2+2p = r^2+r$$



          So $$2p(p+1)=r(r+1)implies rmid 2p(p+1)$$



          If $rmid 2$ then $r=2$ which is impossibile.



          If $rmid p$ then $rleq p$ which is impossibile.



          If $rmid p+1$ then $rleq p+1$ but then $r=p+1$ since $r>p$, so $r=3$ and $p=2$.







          share|cite|improve this answer












          share|cite|improve this answer



          share|cite|improve this answer










          answered Nov 14 at 23:13









          greedoid

          34.5k114489




          34.5k114489






















              up vote
              0
              down vote













              From the given equation
              $$p(p+1)+q(q+1)=r(r+1)$$
              it follows that $p < r$ and $q < r$.



              Next, another inequality which will be useful later . . .



              Claim:$;p+q > r$.



              Proof:
              begin{align*}
              &p(p+1)+q(q+1)=r(r+1)\[4pt]
              implies;&(p^2+q^2)+(p+q)=r(r+1)\[4pt]
              implies;&(p+q)^2+(p+q)>r(r+1)\[4pt]
              implies;&(p+q)(p+q+1)>r(r+1)\[4pt]
              implies;&p+q>r\[4pt]
              end{align*}

              as claimed.



              Returning to the main problem . . .



              First suppose $p=q$.



              Then the given equation reduces to
              $$2p(p+1)=r(r+1)$$
              hence, since $r > p$, it follows that $r|(p+1)$.



              But then $p < r le p+1$, so $r=p+1$, which implies $p=2$, and $r=3$.



              It can be verified that the triple $(p,q,r)=(2,2,3)$ satisfies the given equation.



              Next suppose $p,q$ are distinct.



              Without loss of generality, assume $p < q$.



              Suppose $;p=2$.



              Then from $p < q < r$, we get $qge 3$ and $rge 5$, hence
              begin{align*}
              &p(p+1)+q(q+1)=r(r+1)\[4pt]
              implies;&(2)(2+1)+q(q+1)=r(r+1)\[4pt]
              implies;&r(r+1)-q(q+1)=6\[4pt]
              implies;&(r-q)(q+r+1)=6\[4pt]
              implies;&(q+r+1)mid 6\[4pt]
              implies;&q+r+1le 6\[4pt]
              end{align*}

              contradiction, since $q+r+1ge 3+5+1=9$.



              Hence we must have $p > 2$.



              Since $p+q > r$, it follows that $pnotmid r-q$, and $qnotmid r-p$.
              begin{align*}
              text{Then};;&p(p+1)+q(q+1)=r(r+1)\[4pt]
              implies;&p(p+1)=r(r+1)-q(q+1)\[4pt]
              implies;&p(p+1)=(q+r+1)(r-q)\[4pt]
              implies;&pmid (q+r+1)\[4pt]
              implies;&pmid (p+q+r+1)\[12pt]
              text{and};,&p(p+1)+q(q+1)=r(r+1)\[4pt]
              implies;&q(q+1)=r(r+1)-p(p+1)\[4pt]
              implies;&q(q+1)=(p+r+1)(r-p)\[4pt]
              implies;&qmid (p+r+1)\[4pt]
              implies;&qmid (p+q+r+1)\[12pt]
              text{hence};,&pqmid (p+q+r+1)\[4pt]
              implies;&pqle p+q+r+1\[4pt]
              implies;&pq < p+q+(p+q)+1\[4pt]
              implies;&pq-2p-2q < 1\[4pt]
              implies;&(p-2)(q-2) < 5\[4pt]
              implies;&q-2 < 5\[4pt]
              implies;&q < 7\[4pt]
              implies;&qle 5\[4pt]
              implies;&(p,q)=(3,5)\[4pt]
              implies;&(3)(3+1)+(5)(5+1)=r(r+1)\[4pt]
              implies;&r=6\[4pt]
              end{align*}

              contradiction, since $6$ is not prime.



              Therefore the only solution is $(p,q,r)=(2,2,3)$.






              share|cite|improve this answer



























                up vote
                0
                down vote













                From the given equation
                $$p(p+1)+q(q+1)=r(r+1)$$
                it follows that $p < r$ and $q < r$.



                Next, another inequality which will be useful later . . .



                Claim:$;p+q > r$.



                Proof:
                begin{align*}
                &p(p+1)+q(q+1)=r(r+1)\[4pt]
                implies;&(p^2+q^2)+(p+q)=r(r+1)\[4pt]
                implies;&(p+q)^2+(p+q)>r(r+1)\[4pt]
                implies;&(p+q)(p+q+1)>r(r+1)\[4pt]
                implies;&p+q>r\[4pt]
                end{align*}

                as claimed.



                Returning to the main problem . . .



                First suppose $p=q$.



                Then the given equation reduces to
                $$2p(p+1)=r(r+1)$$
                hence, since $r > p$, it follows that $r|(p+1)$.



                But then $p < r le p+1$, so $r=p+1$, which implies $p=2$, and $r=3$.



                It can be verified that the triple $(p,q,r)=(2,2,3)$ satisfies the given equation.



                Next suppose $p,q$ are distinct.



                Without loss of generality, assume $p < q$.



                Suppose $;p=2$.



                Then from $p < q < r$, we get $qge 3$ and $rge 5$, hence
                begin{align*}
                &p(p+1)+q(q+1)=r(r+1)\[4pt]
                implies;&(2)(2+1)+q(q+1)=r(r+1)\[4pt]
                implies;&r(r+1)-q(q+1)=6\[4pt]
                implies;&(r-q)(q+r+1)=6\[4pt]
                implies;&(q+r+1)mid 6\[4pt]
                implies;&q+r+1le 6\[4pt]
                end{align*}

                contradiction, since $q+r+1ge 3+5+1=9$.



                Hence we must have $p > 2$.



                Since $p+q > r$, it follows that $pnotmid r-q$, and $qnotmid r-p$.
                begin{align*}
                text{Then};;&p(p+1)+q(q+1)=r(r+1)\[4pt]
                implies;&p(p+1)=r(r+1)-q(q+1)\[4pt]
                implies;&p(p+1)=(q+r+1)(r-q)\[4pt]
                implies;&pmid (q+r+1)\[4pt]
                implies;&pmid (p+q+r+1)\[12pt]
                text{and};,&p(p+1)+q(q+1)=r(r+1)\[4pt]
                implies;&q(q+1)=r(r+1)-p(p+1)\[4pt]
                implies;&q(q+1)=(p+r+1)(r-p)\[4pt]
                implies;&qmid (p+r+1)\[4pt]
                implies;&qmid (p+q+r+1)\[12pt]
                text{hence};,&pqmid (p+q+r+1)\[4pt]
                implies;&pqle p+q+r+1\[4pt]
                implies;&pq < p+q+(p+q)+1\[4pt]
                implies;&pq-2p-2q < 1\[4pt]
                implies;&(p-2)(q-2) < 5\[4pt]
                implies;&q-2 < 5\[4pt]
                implies;&q < 7\[4pt]
                implies;&qle 5\[4pt]
                implies;&(p,q)=(3,5)\[4pt]
                implies;&(3)(3+1)+(5)(5+1)=r(r+1)\[4pt]
                implies;&r=6\[4pt]
                end{align*}

                contradiction, since $6$ is not prime.



                Therefore the only solution is $(p,q,r)=(2,2,3)$.






                share|cite|improve this answer

























                  up vote
                  0
                  down vote










                  up vote
                  0
                  down vote









                  From the given equation
                  $$p(p+1)+q(q+1)=r(r+1)$$
                  it follows that $p < r$ and $q < r$.



                  Next, another inequality which will be useful later . . .



                  Claim:$;p+q > r$.



                  Proof:
                  begin{align*}
                  &p(p+1)+q(q+1)=r(r+1)\[4pt]
                  implies;&(p^2+q^2)+(p+q)=r(r+1)\[4pt]
                  implies;&(p+q)^2+(p+q)>r(r+1)\[4pt]
                  implies;&(p+q)(p+q+1)>r(r+1)\[4pt]
                  implies;&p+q>r\[4pt]
                  end{align*}

                  as claimed.



                  Returning to the main problem . . .



                  First suppose $p=q$.



                  Then the given equation reduces to
                  $$2p(p+1)=r(r+1)$$
                  hence, since $r > p$, it follows that $r|(p+1)$.



                  But then $p < r le p+1$, so $r=p+1$, which implies $p=2$, and $r=3$.



                  It can be verified that the triple $(p,q,r)=(2,2,3)$ satisfies the given equation.



                  Next suppose $p,q$ are distinct.



                  Without loss of generality, assume $p < q$.



                  Suppose $;p=2$.



                  Then from $p < q < r$, we get $qge 3$ and $rge 5$, hence
                  begin{align*}
                  &p(p+1)+q(q+1)=r(r+1)\[4pt]
                  implies;&(2)(2+1)+q(q+1)=r(r+1)\[4pt]
                  implies;&r(r+1)-q(q+1)=6\[4pt]
                  implies;&(r-q)(q+r+1)=6\[4pt]
                  implies;&(q+r+1)mid 6\[4pt]
                  implies;&q+r+1le 6\[4pt]
                  end{align*}

                  contradiction, since $q+r+1ge 3+5+1=9$.



                  Hence we must have $p > 2$.



                  Since $p+q > r$, it follows that $pnotmid r-q$, and $qnotmid r-p$.
                  begin{align*}
                  text{Then};;&p(p+1)+q(q+1)=r(r+1)\[4pt]
                  implies;&p(p+1)=r(r+1)-q(q+1)\[4pt]
                  implies;&p(p+1)=(q+r+1)(r-q)\[4pt]
                  implies;&pmid (q+r+1)\[4pt]
                  implies;&pmid (p+q+r+1)\[12pt]
                  text{and};,&p(p+1)+q(q+1)=r(r+1)\[4pt]
                  implies;&q(q+1)=r(r+1)-p(p+1)\[4pt]
                  implies;&q(q+1)=(p+r+1)(r-p)\[4pt]
                  implies;&qmid (p+r+1)\[4pt]
                  implies;&qmid (p+q+r+1)\[12pt]
                  text{hence};,&pqmid (p+q+r+1)\[4pt]
                  implies;&pqle p+q+r+1\[4pt]
                  implies;&pq < p+q+(p+q)+1\[4pt]
                  implies;&pq-2p-2q < 1\[4pt]
                  implies;&(p-2)(q-2) < 5\[4pt]
                  implies;&q-2 < 5\[4pt]
                  implies;&q < 7\[4pt]
                  implies;&qle 5\[4pt]
                  implies;&(p,q)=(3,5)\[4pt]
                  implies;&(3)(3+1)+(5)(5+1)=r(r+1)\[4pt]
                  implies;&r=6\[4pt]
                  end{align*}

                  contradiction, since $6$ is not prime.



                  Therefore the only solution is $(p,q,r)=(2,2,3)$.






                  share|cite|improve this answer














                  From the given equation
                  $$p(p+1)+q(q+1)=r(r+1)$$
                  it follows that $p < r$ and $q < r$.



                  Next, another inequality which will be useful later . . .



                  Claim:$;p+q > r$.



                  Proof:
                  begin{align*}
                  &p(p+1)+q(q+1)=r(r+1)\[4pt]
                  implies;&(p^2+q^2)+(p+q)=r(r+1)\[4pt]
                  implies;&(p+q)^2+(p+q)>r(r+1)\[4pt]
                  implies;&(p+q)(p+q+1)>r(r+1)\[4pt]
                  implies;&p+q>r\[4pt]
                  end{align*}

                  as claimed.



                  Returning to the main problem . . .



                  First suppose $p=q$.



                  Then the given equation reduces to
                  $$2p(p+1)=r(r+1)$$
                  hence, since $r > p$, it follows that $r|(p+1)$.



                  But then $p < r le p+1$, so $r=p+1$, which implies $p=2$, and $r=3$.



                  It can be verified that the triple $(p,q,r)=(2,2,3)$ satisfies the given equation.



                  Next suppose $p,q$ are distinct.



                  Without loss of generality, assume $p < q$.



                  Suppose $;p=2$.



                  Then from $p < q < r$, we get $qge 3$ and $rge 5$, hence
                  begin{align*}
                  &p(p+1)+q(q+1)=r(r+1)\[4pt]
                  implies;&(2)(2+1)+q(q+1)=r(r+1)\[4pt]
                  implies;&r(r+1)-q(q+1)=6\[4pt]
                  implies;&(r-q)(q+r+1)=6\[4pt]
                  implies;&(q+r+1)mid 6\[4pt]
                  implies;&q+r+1le 6\[4pt]
                  end{align*}

                  contradiction, since $q+r+1ge 3+5+1=9$.



                  Hence we must have $p > 2$.



                  Since $p+q > r$, it follows that $pnotmid r-q$, and $qnotmid r-p$.
                  begin{align*}
                  text{Then};;&p(p+1)+q(q+1)=r(r+1)\[4pt]
                  implies;&p(p+1)=r(r+1)-q(q+1)\[4pt]
                  implies;&p(p+1)=(q+r+1)(r-q)\[4pt]
                  implies;&pmid (q+r+1)\[4pt]
                  implies;&pmid (p+q+r+1)\[12pt]
                  text{and};,&p(p+1)+q(q+1)=r(r+1)\[4pt]
                  implies;&q(q+1)=r(r+1)-p(p+1)\[4pt]
                  implies;&q(q+1)=(p+r+1)(r-p)\[4pt]
                  implies;&qmid (p+r+1)\[4pt]
                  implies;&qmid (p+q+r+1)\[12pt]
                  text{hence};,&pqmid (p+q+r+1)\[4pt]
                  implies;&pqle p+q+r+1\[4pt]
                  implies;&pq < p+q+(p+q)+1\[4pt]
                  implies;&pq-2p-2q < 1\[4pt]
                  implies;&(p-2)(q-2) < 5\[4pt]
                  implies;&q-2 < 5\[4pt]
                  implies;&q < 7\[4pt]
                  implies;&qle 5\[4pt]
                  implies;&(p,q)=(3,5)\[4pt]
                  implies;&(3)(3+1)+(5)(5+1)=r(r+1)\[4pt]
                  implies;&r=6\[4pt]
                  end{align*}

                  contradiction, since $6$ is not prime.



                  Therefore the only solution is $(p,q,r)=(2,2,3)$.







                  share|cite|improve this answer














                  share|cite|improve this answer



                  share|cite|improve this answer








                  edited Nov 14 at 23:52

























                  answered Nov 14 at 23:37









                  quasi

                  35.9k22562




                  35.9k22562






























                       

                      draft saved


                      draft discarded



















































                       


                      draft saved


                      draft discarded














                      StackExchange.ready(
                      function () {
                      StackExchange.openid.initPostLogin('.new-post-login', 'https%3a%2f%2fmath.stackexchange.com%2fquestions%2f2056120%2fhow-do-i-find-all-prime-solutions-p-q-r-of-the-equation-displaystyle-pp1%23new-answer', 'question_page');
                      }
                      );

                      Post as a guest















                      Required, but never shown





















































                      Required, but never shown














                      Required, but never shown












                      Required, but never shown







                      Required, but never shown

































                      Required, but never shown














                      Required, but never shown












                      Required, but never shown







                      Required, but never shown







                      Popular posts from this blog

                      Plaza Victoria

                      In PowerPoint, is there a keyboard shortcut for bulleted / numbered list?

                      How to put 3 figures in Latex with 2 figures side by side and 1 below these side by side images but in...